Difference between revisions of "MIE 2016/Problem 2"

(Created page with "===Problem 2=== The following system has <math>k</math> integer solutions. We can say that: <math>\begin{cases}\frac{x^2-2x-14}{x}>3\\\\x\leq12\end{cases}</math> (a) <math>...")
 
(Problem 2)
Line 1: Line 1:
===Problem 2===
+
==Problem 2==
 
The following system has <math>k</math> integer solutions. We can say that:
 
The following system has <math>k</math> integer solutions. We can say that:
  

Revision as of 21:20, 7 January 2018

Problem 2

The following system has $k$ integer solutions. We can say that:

$\begin{cases}\frac{x^2-2x-14}{x}>3\\\\x\leq12\end{cases}$


(a) $0\leq k\leq 2$

(b) $2\leq k\leq 4$

(c) $4\leq k\leq6$

(d) $6\leq k\leq8$

(e) $k\geq8$

Solution 2

See Also